CFP Scan Exam

Ace your homework & exams now with Quizwiz!

Assume you bought 100 shares of XYZ stock for $60 per share with an initial margin of 50% and a 30% maintenance margin. What will be the amount of the margin call if the stock drops to $40 per share? A) $800 B) $200 C) $0 D) $1,800

B) $200 Market value= $4,000 ($40 × 100 shares) Loan= ($3,000) (50% of $6,000) Current equity= $1,000 Required equity= $1,200 Cash required= $200 ($1,200 − $1,000)

A simplified employee pension (SEP) plan: 1. uses an IRA as a funding vehicle. 2. must follow the rule regarding non-discriminatory contributions. 3. may include a loan provision. 4. requires a specific employer contribution each year. A) 1, 2, and 3. B) 2, 3, and 4. C) 4 only. D) 1 and 2.

D) 1 and 2. A SEP uses an IRA as a funding vehicle. Employer contributions are discretionary, but any contributions that are made must be nondiscriminatory. Because SEPs are funded with IRAs, loan provisions are not permitted.

Which of the following statements regarding a self-canceling installment note (SCIN) is(are) CORRECT? 1. To be effective, a self-canceling installment note must reflect a risk premium to compensate the seller for the possibility of cancellation. 2. A seller who accepts a self-canceling installment note may not require security for the note without jeopardizing the estate planning advantages of the SCIN. 3. At the seller's death, the present value of any remaining self-canceling installment note balance is excluded from the seller's gross estate. 4. A self-canceling installment note is a debt that ordinarily is extinguished at the seller's death. A) 1 and 2 B) 1, 3, and 4 C) 1 and 4 D) 2 and 3

B) 1, 3, and 4 All of these statements are correct except that a SCIN may be collaterized.

Ed is employed by ABC Trucking, where he is covered by a health insurance plan, a contributory long-term disability insurance plan, and a pension plan. Ed pays $40 per quarter and ABC pays $60 per quarter for a monthly disability benefit of $1,000 following a 60-day elimination (waiting) period. During the current year, Ed was disabled for 5 months. How much, if any, of the disability insurance benefits are taxable to Ed in the current year? A) $1,200 B) $1,800 C) $0 D) $3,000

B) $1,800 After satisfying a 60-day elimination period, Ed received 3 months of disability insurance benefits of $1,000 per month. Because Ed pays 40% of the premiums and the company pays 60%, Ed is taxed on 60% of the benefits. The taxable portion of the benefits is $1,800 ($3,000 × 60%).

When conducting an interview with a client, using open-ended questions is important to gather information. Which of the following questions would be considered open-ended? 1. Do you have an IRA? 2. What are your financial goals in terms of retirement? 3. What are your feelings about investing in the stock market? A) 1 only B) 2 and 3 C) 1, 2, and 3 D) 2 only

B) 2 and 3 Open-ended questions allow planners to get detailed information not always obtained with closed-ended questions that require only a "yes" or "no" answer. Statement 1 is incorrect because this is a closed-ended question; i.e., it requires only a "yes" or "no" answer. Statements 2 and 3 are correct; these are open-ended questions because they require the client to answer in her own words.

Mary Sue's salaries from 2008-2022 are shown below. What is the maximum annual compensation that may be used to determine Mary Sue's annual benefit under a traditional defined benefit pension plan for 2022? Salaries: 2008-2016 $65,000 2017 $100,000 2018 $120,000 2019 $90,000 2020 $125,000 2021 $110,000 2022 $90,000 A) $111,667 B) $108,333 C) $96,666 D) $140,000

A) $111,667 In a defined benefit pension plan, the maximum annual benefit is limited to the lesser of $245,000 (2022) or the participant's compensation averaged over the three highest consecutive earnings years. In this situation, Mary Sue's highest three consecutive years of compensation are $120,000 + $90,000 + $125,000 = $335,000 ÷ 3 years = $111,667.

Which of the following is included in AMTI for calculating the alternative minimum tax for an individual taxpayer? 1. Excess of depletion deduction over the adjusted basis. 2. Excess of fair market value above the exercise price if exercised for freely transferable ISOs. 3. Net appreciation on long-term capital gain property donated to a public charity. A) 1 and 2 B) 2 only C) 1 and 3 D) 1 only

A) 1 and 2 Excess of the depletion deduction over the adjusted basis, and excess of the FMV above the exercise price if exercised for freely transferable ISOs are both included in AMTI.

On January 10 of the current year, Mark sold stock with an adjusted tax basis of $6,000 to his son Les for $4,000 (fair market value). On July 31 of the next year, Les sold the same stock for $5,000 in a bona fide arms-length transaction to Sara, who is unrelated to Les or Mark. What is the proper tax treatment for these transactions? A) Neither Mark nor Les has a recognized gain or loss in either year. B) Les has a recognized gain of $2,000 in the current year. C) Les has a recognized gain of $1,000 in the next year. D) Mark has a recognized loss of $2,000 in the current year.

A) Neither Mark nor Les has a recognized gain or loss in either year. This is an application of the related party rules. Neither Mark nor Les has a recognized gain or loss in either year. Mark has a $2,000 realized loss in the first year, but cannot recognize it because of the related party rule. Mark forever loses the ability to take a deduction for the loss because it is the result of a related party transaction. Les has a realized gain of $1,000 in the second year. He can reduce his gain by Mark's loss (up to the amount of gain). Les has no gain or loss in the second year. The remaining $1,000 loss is no longer available to either of them.

In the current year, Pamela, who is single, had the following capital gains and losses: Short-term capital gains $40,000 Short-term capital losses $32,000 Long-term capital gains $15,000 Long-term capital losses $27,000 What is Pamela's net capital gain or loss for the current year, and how is it treated on her current year's tax return? A) She has a $3,000 deductible long-term capital loss with a $1,000 long-term capital loss carryover. B) She has a short-term capital gain of $8,000 and a $3,000 long-term capital loss with a $9,000 carryover. C) She has a short-term capital gain of $8,000 and a $12,000 long-term capital loss. D) She has a net capital loss of $4,000, fully deductible in the current year.

A) She has a $3,000 deductible long-term capital loss with a $1,000 long-term capital loss carryover. Pamela has a $3,000 deductible LTCL with a $1,000 LTCL carryover. First, net the long term gains and losses: $27,000 LTCL − $15,000 LTCG = $12,000 LTCL. Then, net the short-term gains and losses: $40,000 STCG − $32,000 STCL = $8,000 STCG. Net the $12,000 LTCL and the $8,000 STCG = $4,000 LTCL. Subtract the maximum deductible LTCL of $3,000 from the net $4,000 LTCL = $1,000 carryover.

Which of the following employee(s) is(are) highly compensated for qualified plan nondiscrimination testing purposes in the current year? 1. Stephen, a 6% owner of an incorporated law firm 2. Franklin, who earned $145,000 last year and he was the top-paid employee 3. Jerome, whose salary was the 10th highest of 50 employees and who earned $75,000 last year 4. Margo, a corporate vice president of marketing and 1% owner of the company, whose salary last year was $68,000 A) 1 and 2 B) 1, 3, and 4 C) 2 only D) 1, 2, and 4

A) 1 and 2 Stephen and Franklin are highly compensated for qualified plan purposes in the current year. A highly compensated employee is one who was a greater than 5% owner of the employer at any time during the current year or preceding year, or for the preceding year, had compensation greater than $135,000 (2022).

Which of the following are characteristics of property owned as tenants by the entirety? 1. Can only be owned by spouses. 2. Each owner has an equal ownership interest in the property. 3. Transfer of property does not require the consent of the other owner. 4. Includes a right of survivorship. A) 1, 2, and 3. B) 1, 2, and 4. C) 4 only. D) 1 and 3.

B) 1, 2, and 4. Tenants by the entirety can only be owned by spouses. Each spouse holds an equal joint interest in the property. Any transfer of the property requires the approval of both spouses acting as one. Upon the death of one spouse, the property passes to the surviving spouse by right of survivorship.

On January 15, of last year, Tim transferred property to a trust, which he retained the right to revoke. The trust pays Amy 5% of the trust assets valued annually for her life, with the remainder to be paid to a qualified charity. What type of arrangement did Tim create? A) CRUT. B) None of these. C) CRAT. D) CLAT.

B) None of these. This trust has some of the features of a charitable remainder unitrust (CRUT), but it does not qualify as a CRUT because it was not irrevocable when it was created.

William is a 17-year-old high school student who has $3,300 of investment income from mutual funds and $5,000 of earnings from a part-time job. His parents claim him as a dependent. How much of William's income is taxed at his parents' marginal tax rate? A) $0 B) $2,200 C) $1,000 D) $3,200

C) $1,000 Under the kiddie tax rules, William's unearned income in excess of $2,300 (2022) will be taxed at his parents' marginal tax rate.

Beth, who is single, had an adjusted gross income of $85,000 in the current year and experienced losses of $30,000 from rental real estate in which she was an active participant. Beth is the sole owner of the property. How much of the real estate loss can she deduct against earned income this year if she otherwise qualifies? A) $15,000. B) $12,500. C) $25,000. D) $3,000.

C) $25,000. Individuals can deduct up to $25,000 of rental real estate losses against active and portfolio income. However, 2 tests must be met to qualify for this exception. One test is active participation in the activity (participates in management decisions). The second test is ownership of 10% or more (in value) of all interests in the activity during the taxable year. In addition, the $25,000 offset allowance is reduced by 50% of AGI in excess of $100,000 with a complete phaseout at $150,000 AGI. Therefore, there is a loss of $1 for every $2 of AGI greater than $100,000. The loss is deducted from the $25,000 maximum available. In this situation, Beth meets both of the qualification tests and her AGI is below $100,000.

Matt gifts Jim securities in the current year. Matt's adjusted basis for the securities is $48,000, and the fair market value is $40,000. Matt pays $2,000 in gift tax. What is Jim's basis for the stock for gain and for loss? A) $40,000 for gain and $40,000 for loss. B) $0 for gain and $0 for loss. C) $48,000 for gain and $40,000 for loss. D) $50,000 for gain and $42,000 for loss.

C) $48,000 for gain and $40,000 for loss. Because the FMV of the property at the time of the gift was less than Matt's adjusted basis, Jim's basis depends on whether he sells the property for a gain or for a loss. His basis for determining gain is Matt's adjusted basis, which is $48,000. His basis for determining loss is the FMV of $40,000. Because this is a gift of loss property, none of the gift tax paid will be added to Jim's basis.

On January 15 of the current year, Kermit transfers property to an irrevocable trust. The trust is to pay Holly 5% of the trust assets valued annually each year for her life, with the remainder to be paid to a qualified charity. On September 1 of the next year, Kermit dies. Which of the following statements is(are) CORRECT? 1. This is a charitable remainder unitrust (CRUT). 2. Kermit receives a charitable income tax deduction equal to the present value of the remainder interest in the current year. 3. The value of the trust assets will be included in Kermit's gross estate. A) 1 only. B) 1 and 3. C) 1 and 2. D) 2 and 3.

C) 1 and 2. This is a charitable remainder unitrust (CRUT) because it pays Kermit a percentage of the trust assets as valued each year. Because the trust is a charitable remainder trust, Kermit receives a charitable income tax deduction in the current year equal to the present value of the remainder interest. The trust assets are not included in Kermit's gross estate because Kermit transferred the assets to the trust before he died and the trust was irrevocable.

Bob works for the ABC Company. In fact, he owns 4% of ABC. His 401(k) balance was $450,000 as of December 31 of last year. He turns 73 this year and plans on retiring on his birthday. Bob will take $15,000 out of his 401(k) this year on December 31. The life expectancy for someone age 73 is 24.7 years. Which of the following statements regarding Bob's required minimum distributions is CORRECT? A) Bob's distribution of $15,000 is sufficient to meet his required minimum distribution requirement. B) Bob will incur a penalty because he failed to take the required minimum distribution by December 31 of the current year. C) Bob will not have a penalty for the current year, but he will need to withdraw the balance of his required minimum distribution for the current year by April 1 of next year. D) Bob is not subject to the required minimum distribution rules.

C) Bob will not have a penalty for the current year, but he will need to withdraw the balance of his required minimum distribution for the current year by April 1 of next year. Bob is greater than 72 and owns less than 5% of the ABC Company. Thus, he can delay RMDs until April 1 of the year following the year he retires. If he retires this year, his required beginning date (RBD) will be April 1 of next year. Based on his 401(k) of $450,000 and a life expectancy of 24.7 years, the required minimum distribution is $18,219 ($450,000 ÷ 24.7). Bob plans to take a $15,000 distribution this year, but he will not incur a penalty tax unless he fails to withdraw the remaining $3,219 by April 1 of next year.

A client consults a financial professional for help in formulating an estate plan. The client is in poor health and expects to die within the next 3 to 4 years. He has a large estate and would like to begin taking steps to reduce any estate tax that might be due at his death. The client is a widower with 1 adult daughter. The client owns the following property in his name alone: A life insurance policy insuring his own life, with a death benefit of $5 million A personal residence with a market value of $6 million A brokerage fund with a balance of $10 million Which of the following steps should the client implement first to meet his objectives? A) Gift the residence to his daughter. B) Transfer his residence to an irrevocable living trust. C) Transfer ownership of the life insurance policy to his daughter. D) Add his daughter's name to the brokerage account as JTWROS.

C) Transfer ownership of the life insurance policy to his daughter. The proceeds of the life insurance policy will be included in the client's gross estate if the client owns the policy when he dies or if he transferred ownership of the policy within 3 years before his death. Given the client's poor health and short life expectancy, the most urgent step is to transfer ownership of the policy to his daughter. If the client survives for 3 years after the transfer, the $5 million in death benefits will be removed from his gross estate. The 3-year rule would not apply to a gift of the residence to an irrevocable trust or to the daughter directly, so making either of these transfers is less urgent. Adding his daughter's name to the brokerage account as JTWROS would not reduce the client's gross estate because he supplied all of the consideration for the account.

Steve began his professional corporation single practitioner CPA firm 38 years ago. He worked profitably as a sole practitioner for the full 38 years until retiring December 31 of last year at his full retirement age. On January 1 of this year, he sold his practice for $400,000 to be received in 4 equal, annual payments of $100,000, beginning on the date of sale and continuing each January 1 for the next 3 years. Is Steve eligible for Social Security retirement benefits during this year and why? A) No, he is not considered retired for Social Security purposes until the installment payments are complete. B) Yes, he is entitled to benefits, but they will be reduced because of the payments from the sale of his practice. C) Yes, he is eligible because he retired at his full retirement age and is fully insured. D) No, even though he retired at his full retirement age, the proceeds from the sale of his practice will delay his receiving Social Security retirement benefits.

C) Yes, he is eligible because he retired at his full retirement age and is fully insured. Social Security benefits may be reduced for individuals under full retirement age with earned income above certain levels. The proceeds of the installment sale are not considered earned income for Social Security retirement benefit purposes. Therefore, Steve's benefits will not be reduced. He has attained his full retirement age and is fully insured. Note: The reduction of Social Security benefits based on earned income no longer applies once an individual reaches full retirement age. Because Steve retired at his full retirement age, his benefits would not be reduced, even if he had earned income.

Calculate the maximum benefit that may be funded for in the current year for Winona, who participates in her employer's traditional defined benefit pension plan. Her highest three consecutive years' salaries are shown below. Winona will have more than 10 years of service with the same employer at retirement. Highest Salaries Current year (−2) $ 90,000 Current year (−1) $100,000 Current year $110,000 A) $90,000 B) $135,000 C) $120,000 D) $100,000

D) $100,000 The maximum annual benefit under a traditional defined benefit pension plan is the lesser of $235,000 (2022) or the average of the participant's compensation over the three consecutive highest earning years. In this case, the maximum benefit is $100,000 [($90,000 + $100,000 + $110,000) ÷ 3].

John, age 55, is divorced and retired. He has the following liquid assets on deposit at Allworld Bank, an FDIC-insured financial institution: Account Ownership Balance Certificate of deposit John $225,000 Savings account Joint with son $70,000 Rollover traditional IRA John $150,000 Checking account John $80,000 What amount is insured by the FDIC? A) $325,000 B) $250,000 C) $525,000 D) $470,000

D) $470,000 The FDIC insures separate legal categories of accounts. As a result, the IRA will be insured for $150,000, but can be insured up to $250,000 as the balance increases. The individual accounts (checking and CD owned by John are aggregated and are insured up to $250,000 in total. The joint account is insured for $70,000.

Which of the following pension plans must be covered by Pension Benefit Guarantee Corporation (PBG) insurance? 1. Cash balance pension plan. 2. Money purchase pension plan. 3. Target benefit pension plan. 4. Traditional defined benefit pension plan. A) 2 and 3. B) 1, 2, 3, and 4. C) 3 only. D) 1 and 4.

D) 1 and 4. Only defined benefit pension plans (including cash balance pension plans) are covered by the PBGC.

Mama Mason's Inc., a regular C Corporation, is considering the adoption of a qualified retirement plan. The company has had fluctuating cash flows in the recent past, and such fluctuations are expected to continue into the future. The average age of nonowner employees is 24, and the average number of years of service is 3, with a high of 4 and a low of 1. Approximately 25% of the 12-person labor force turns over each year. The 2 owners earn approximately 2/3 of the covered compensation. Which is the most appropriate plan for Mama Mason's? A) Traditional defined benefit pension plan. B) Money purchase pension plan. C) Target benefit pension plan. D) Profit-sharing plan.

D) Profit-sharing plan. Because of the fluctuating cash flows, the most appropriate plan is a profit-sharing plan. Each of the other options given require a mandatory annual contribution.

Ryan and Kim have a 2-year-old son, Michael. One of their goals is to begin saving now for Michael's high school tuition at River Oaks Academy. After analyzing Ryan and Kim's financial statements and other relevant information, you conclude that they should save $2,000 at the beginning of the year for the next 12 years. Which of the following education planning vehicles is the most appropriate recommendation for Ryan and Kim? A) Certificates of deposit (CDs) B) Series EE savings bonds C) Lifetime Learning Credit D) Section 529 plan

D) Section 529 plan As a result of the Tax Cuts and Jobs Act (TCJA), elementary and secondary school tuition up to $10,000/year is considered a qualified education expense within Section 529 plans. Certificates of deposit (CDs) are not the best investment option for Ryan and Kim because they offer a low rate of return that will not keep up with the pace of education inflation. Funds saved in Series EE savings bonds may not be used for high school expenses. The Lifetime Learning Credit is not a savings vehicle; it is a tax credit that may be used for post-secondary education.

Which of the following is appropriate justification for selling a stock short? A) To cut losses on a long position B) To seek a modest potential reward with limited risk C) To benefit from a rise in the price of the stock D) To benefit from a decline in the price of the stock

D) To benefit from a decline in the price of the stock Selling short does not reduce the risk of a long position—the investor is selling borrowed, not owned, stock. The appropriate time to sell short is when one anticipates a drop in the stock's price. The investor wants to sell at a high price and buy later at a lower price. Both the reward and risk potential of selling short are high. If the stock price falls dramatically, the investor can reap a large gain. If it rises dramatically, the investor can lose a large amount of money.

Three years ago, Carol invested $10,000 in Uncle Sam Mutual Fund, (which only invests in U.S. Treasuries). The fund paid interest of $700 the first year, $900 the second year, and $0 the third year. Interest for all years was not distributed, but instead, immediately reinvested. The value of the account as of January 1 of the current year is $12,000. Assume capital gains are taxed at 15%, her ordinary income is taxed at 32%, and her state tax rate is 4% for ordinary income and capital gains. If she sells her shares today, what is the value of this investment at the end of the holding period after deducting all taxes? A) $11,412 B) $11,924 C) $11,856 D) $11,300

A) $11,412 Because Carol reinvested the interest, she can add the interest to her cost basis when she sells the shares. Carol paid $224 in federal income taxes in year 1 ($700 × 0.32). She paid $288 in income federal taxes in year 2 ($900 × 0.32). Interest from U.S. Treasuries is subject to federal income tax but not state income tax in the year it is earned. This is true regardless of if the interest was actually distributed to the client. Because her broker reinvested the entire interest and did not withhold any tax, her cost basis for the account is $11,600 ($10,000 + $700 + $900). As such, the tax due would have been reported and paid on her Form 1040 for those years. She sold the fund for $12,000. Therefore, she has a capital gain of $400. Treasury interest is exempt from state tax, while capital gains on Treasuries are usually subject to state tax. Therefore, her capital gains tax is ($400 × 0.15) + ($400 × 0.04) = $60 + $16 = $76. Her total taxes paid is $224 + $288 + $76 = $588. The value of her investment for the holding period after taxes is $12,000 − $588 = $11,412.

John and Patty have newborn twin boys. John, a financial professional, understands the importance of planning ahead and wants to begin saving for their boys' college education. John and Patty estimate the annual tuition to be about $25,000 (in today's dollars) per child. Although inflation has averaged 4%, the cost of education has been increasing at an average rate of 6% per year. To fund 4 years of college for each boy beginning in 18 years, how much must John and Patty save at the end of each year for the next 18 years? Assume that John and Patty invest in equities and can earn an after-tax annual return of 9%. (Round to the next multiple of $100) A) $13,300 B) $4,600 C) $9,200 D) $6,700

A) $13,300 Step 1: Calculate the FV of $25,000 in 18 years at 6% inflation. PV = 25,000 N = 18 I/YR = 6 Solve for FV = 71,358, or $71,358, per child × 2 boys = $142,717 first year tuition for both sons Step 2: Determine the total inflation adjusted value during the time the boys are in college for 4 years. BEG mode PMT = 142,717 I/YR = 2.8302 Inflation-adjusted rate of return [((1.09 ÷ 1.06) − 1) × 100] N = 4 FV = 0 Solve for PV = 547,730, or $547,730 Step 3: Discount the inflation adjusted value for 18 years at a 9% return to determine how much must be saved at the end of each year for the next 18 years. FV = 547,730 N = 18 I/YR = 9 PV = 0 Solve for PMT = 13,261, rounded to $13,300

Rob built a house several years ago in New Orleans. The replacement value has increased to $200,000. Rob originally purchased insurance on the house under a homeowners broad form HO-2 policy in the amount of $150,000. The policy contains an 80% coinsurance clause. The roof of the house has been damaged by fire. Since the home was built, the roof had depreciated by 25%. The cost to replace the roof will be $20,000. How much will Rob collect from his insurance policy? A) $15,000 less the deductible B) $20,000 less the deductible C) $18,750 less the deductible D) $0

A) $15,000 less the deductible The insurer will pay $18,750 less the deductible. Rob does not meet the coinsurance requirement so the insurer will pay as follows: Did carry:$150,000= $150,000= 93.75% Should have carried:($200,000 × 0.80)=$160,000 The loss is $20,000 (replacement value).Insurer pays 93.75% of each partial loss.Rob pays 6.25% of each partial loss.$20,000 × 0.9375 = $18,750 (less the deductible)

Sally purchased a single premium deferred annuity (SPD in 1990 at a cost of $42,000. Her lifetime annuity distributions of $833.33 per month will begin on September 1st of the current year, at which time her life expectancy will be 21 years. How much must Sally include in her gross income from this SPDA in the current year? (Round to the nearest dollar) A) $2,667 B) $8,000 C) $2,000 D) $667

A) $2,667 Sally must include $2,667 in her gross income in the current year. Before the annuitant reaches the age of projected life expectancy, each payment from a fixed annuity is considered partially a return of basis and partially as taxable income as determined by an exclusion ratio. Therefore, $42,000 (cost basis) divided by $210,000 (total expected benefits over 21 years) equals 20% exclusion ratio. This means 20% of the payments in the current year ($3,333 × 20%), or $667, is excluded from gross income. The remaining 80%, or $2,667, must be included in gross income.

RST stock does not trade on a regular basis. Carl makes a gift of the stock on Thursday, October 5 and the most recent trades for RST stock are as follows: Date Price October 2 $240 October 4 $180 October 9 $265 October 10 $290 What is the value that should be used for the federal gift tax return? A) $208.33. B) $243.75. C) $222.50. D) $180.00.

A) $208.33. If the stock is not traded on the date of gift, the gift tax value of the stock is the price following the date of the gift multiplied by the number of days from the stock trade before the date of the gift. Added to this is the stock price directly preceding the gift date multiplied by the number of days (trading days) between the date of the gift and the next trading day. This sum should be divided by the sum of the days before and after the date of the gift. In this case, [(1 × $265) + (2 × $180)] ÷ 3 days (2 + 1) = $208.33.

Darby is single and has two dependents. His divorce was final in 2021 and financial records show the following were received by Darby in the current year: Gift from a friend $12,000 Cash dividends received from domestic common stock $1,200 Prize won in state lottery $1,000 Salary from employer $35,000 Child support received from ex-spouse $6,000 Alimony received from ex-spouse $12,000 Long term capital loss $5,000 What is Darby's adjusted gross income (AGI) for the current year? A) $34,200. B) $44,200. C) $48,200. D) $43,200.

A) $34,200. Darby's AGI for the current year is $34,200 calculated as follows: cash dividends $1,200 + lottery prize $1,000 + salary $35,000 − LTCL $3,000 (maximum allowed in one tax year). Both the gift from a friend and the child support are excluded from gross income. Alimony received is also excluded from income because the divorced was finalized after December 31st, 2018. Darby having dependents means filing as a head of household rather than filing as single.

Clarence, a CFP® professional, is meeting with his new client, Stephanie, to discuss her financial goals. She brought with the financial documents requested on the data-gathering form Clarence emailed her earlier in the week. Clarence started the meeting by asking Stephanie general background questions, listening carefully to her responses, and taking detailed notes. Then, he asked a series of open-ended questions to elicit additional information about her goals and expectations. Stephanie told him she is an avid reader, volunteers for a local conservation group, and works at the family's lakefront business during the summer. Which of the following represent qualitative data that Clarence obtained during this step of the financial planning process? 1. Stephanie's risk tolerance level. 2. Stephanie's income tax returns from the previous three years. 3. Financial goals and expectations that Stephanie reported to Clarence. 4. Retirement account statements. A) 1 and 3 B) 3 and 4 C) 1, 3, and 4 D) 2 and 4

A) 1 and 3 Qualitative, or subjective, data represent the client's feelings, opinions, and attitudes. Examples include financial goals and risk tolerance level. Conversely, quantitative, or objective, data are measurable or expressed as a quantity or number. Both quantitative and qualitative data are collected during step one of the financial planning process, "Understanding the Client's Personal and Financial Circumstances."

Which of the following types of student financial aid is(are) need-based? 1. Subsidized Stafford Loan 2. Unsubsidized Stafford Loan 3. Parent Loan for Undergraduate Students (PLUS) Loan 4. Pell Grant A) 1 and 4 B) 1, 3, and 4 C) 1 only D) 2 and 3

A) 1 and 4 Subsidized Stafford Loans and Pell Grants are need-based types of financial aid.

Which of the following statements regarding the skilled nursing facility benefit under Medicare Part A is(are) CORRECT? After 100 days of coverage in a benefit period, the patient must pay the entire cost of remaining in the facility. The skilled nursing facility benefit pays the entire cost of the first 30 days while the patient is in the facility. The skilled nursing facility benefit pays for custodial care received in a nursing home. A) 1 only B) 2 and 3 C) 1, 2, and 3 D) 1 and 3

A) 1 only Statement 2 is incorrect; the skilled nursing facility pays the entire cost of the patient's stay in a skilled nursing facility for only the first 20 days. Statement 3 is incorrect. The skilled nursing facility benefit provides no coverage for custodial care.

Which of the following employed individuals may make a deductible contribution to a traditional IRA for 2022? Person Tax Filing Status MAGI Covered by an Employer-Sponsored Retirement Plan 1. Julie Single $60,000 Yes 2. Jerry Single $250,000 No 3.Buffy Single $20,000 Yes 4. Marilyn MFJ $100,000 Yes A) 1, 2, 3, and 4 B) 2 and 3 C) 1, 3, and 4 D) 2 only

A) 1, 2, 3, and 4 Each of these individuals may make deductible contributions to a traditional IRA in 2022. For 2022, eligible taxpayers may contribute a maximum of $6,000 for themselves and $6,000 for their spouses (working or nonworking), increased to $7,000 for those age 50 or older. For taxpayers who are not active participants (and whose spouses are not active participants) in a qualified plan, SEP, SARSEP, SIMPLE, or Section 403(b) plan, contributions to a traditional IRA are fully deductible, regardless of the taxpayer's MAGI. For taxpayers who are active participants in a qualified plan, SEP, SARSEP, SIMPLE, or Section 403(b) plan, the deduction for traditional IRA contributions is limited (or eliminated when a taxpayer's adjusted income reaches phaseout levels of $68,000-$78,000 for single and $109,000-$129,000 for married filing jointly in 2022. If one spouse is an active participant in an employer-sponsored plan and one spouse is not an active participant, the active participant spouse is subject to the $109,000−$129,000 phaseout threshold and the non-active participant spouse is subject to a $204,000−$214,000 phaseout threshold (2022).

Which of the following statements regarding the income tax treatment of involuntary conversion of real property is(are) CORRECT? 1. Gain may be deferred if the taxpayer reinvests the amount realized from the converted property in another property. 2. The period for reinvestment is 2 years from the end of the year in which the realization took place for conversion events caused by nature (e.g., fire, earthquake). 3. The reinvestment period for conversion acts caused by government (eminent domain) is 3 years from the end of the year in which realization of the conversion took place. 4. If the conversion was into cash, the nonrecognition treatment is mandatory, not elective. A) 1, 2, and 3. B) 1 only. C) 3 and 4. D) 1 and 2.

A) 1, 2, and 3. Gain may be deferred if the taxpayer reinvests the amount realized from the converted property in another property. The period for reinvestment is 2 years from the end of the year in which the realization took place for conversion events caused by nature. The reinvestment period for conversion acts caused by government/eminent domain is 3 years from the end of the year in which realization of the conversion took place. If the conversion is into cash, nonrecognition is elective.

Which of the following statements regarding qualified disclaimers is(are) CORRECT? 1. They must be in writing. 2. The disclaiming party cannot have previously benefited from the interest being disclaimed. 3. They generally must be made within 6 months of the creation of the interest. 4. The disclaiming party cannot direct the disclaiming interest to other parties. A) 1, 2, and 4 B) 1, 2, 3, and 4. C) 3 only. D) 3 and 4.

A) 1, 2, and 4 All of these statements are correct except a qualified disclaimer generally must be made within 9 months of the creation of the interest.

Which of the following items would NOT be included on a statement of financial position? 1. Adjusted tax basis of a real estate investment 2. Investment income 3. Fair market value of automobiles 4. Mortgage payments A) 1, 2, and 4 B) 1 and 2 C) 2 and 4 D) 3 only

A) 1, 2, and 4 The adjusted tax basis of a real estate investment would not be included on a statement of financial position. Only the fair market value (FMV) of the real estate investment would be included as an asset. Mortgage principal balances, not mortgage payments, are included on the statement of financial position. Investment income would be reported on a statement of cash flows, not a statement of financial position.

Which of the following statements regarding loans from qualified plans is(are) CORRECT? 1. As a general rule, the maximum loan amount cannot exceed $50,000. 2. The limit on the term of a loan is generally 5 years. 3. Designated employee Roth contributions to a Roth 401(k) are not available for loans. 4. Generally, loans to a 100% owner employee are permissible as long as they are not discriminatory. A) 1, 2, and 4. B) 2, 3, and 4. C) 1 only. D) 2 only.

A) 1, 2, and 4. As a general rule, the maximum qualified plan loan amount is $50,000. The maximum term on qualified plan loans is usually 5 years. Loans to a 100% owner are permissible as long as they are not discriminatory. Designated Roth contributions are available for retirement plan loans because they are vested by definition because they come from the worker's salary. A Roth conversion is not vested by definition. Roth conversion money is vested according to the person's years of service. Of course, no type of IRA may offer a loan (traditional IRA, Roth IRA, SEP, or SIMPLE IRA).

The Severn Partnership has 4 partners. The partners have entered into a binding buy-sell agreement that requires the surviving partners to purchase the partnership interest of the first partner to die. The partners used a cross-purchase agreement, but the agreement remains unfunded. If the partners decide to use life insurance as a funding vehicle, how many policies will be required? A) 12. B) 4. C) 16. D) 1.

A) 12. With a cross-purchase buy-sell agreement, each partner purchases an insurance policy on the other partners. Each of the 4 partners purchases an insurance policy on the other 3 partners (4 × 3) for a total of 12 policies.

Vicki, age 62, has a net worth of $540,000. Her home has a fair market value of $250,000. She recently had a stroke and is paralyzed. Vicki has 2 children and 6 grandchildren. As her financial planner, which of the following would you recommend? 1. Transfer the house to an irrevocable trust in an effort to qualify for Medicaid immediately. 2. Transfer both the house and contents to a revocable living trust to avoid probate. 3. Give 1 of the adult children a durable power of attorney for health care. 4. Gifts of the annual exclusion amount to every child and grandchild each year. A) 2 and 3. B) 3 and 4. C) 3 only. D) 1, 2, and 4.

A) 2 and 3. Transferring both the house and contents to a revocable living trust to avoid probate and giving 1 of the adult children a durable power of attorney for health care are good recommendations. Because of the look-back period, transferring the house to an irrevocable trust in an effort to qualify for Medicaid may result in Vickie being subject to a delay before qualifying for Medicaid. Gifts of the annual exclusion amount to every child and grandchild each year would divert resources needed for Vickie's care. With her poor health and low level of assets, it is not advisable for Vickie to reduce her available resources.

An investor invests $1,000 into the shares of the Stratford Growth and Income Fund, an open-end investment company registered under the Investment Company Act of 1940. On the purchase application, the investor checked the boxes signifying that dividends were to be paid out in cash and capital gains were to be reinvested. During year, the fund pays dividends of $20 and distributes a $250 capital gain. At the end of the year, the fund's value is $1,300. The total return to this investor was A) 32% B) 27% C) 25% D) 30%

A) 32% Total return includes the income from dividends or interest plus any capital appreciation over a given time period, usually one year. For this question, the ending balance of $1,300 includes the capital gain distribution of $250. As a result, only add $20 to the ending balance of $1,300 and subtract $1,000 from this sum to arrive at the total gain of $320. Finally, divide $320 by the beginning value of $1,000 to find the total return of 32%.

Jane comes to you for financial advice. She recently invested $100,000 in ABC Mutual Fund. The fund has an average rate of return of 17% and a standard deviation of 8.5%. The fund also has an alpha of 6.3, a beta of 1.15, and coefficient of determination of 0.80. What is the probability of ABC Mutual Fund's return next year will be equal to or greater than 8.5%? A) 84% B) 34% C) 16% D) 50%

A) 84% Given an average rate of return of 17% and a standard deviation of 8.5%: - 50% probability the returns will be more than 17% and a 50% probability they will be less than 17%. - 68% probability the returns will be between 8.5% and 25.5%. As a result, there is an 84% probability returns will be equal to or greater than 8.5% (the 50% probability returns will be more than 17%, plus the 34% probability they will be between 8.5% and 17%).

Which of the following statements regarding employee elective deferrals under a Section 401(k) plan is(are) CORRECT? A) These contributions are subject to FICA and FUTA but not to federal income tax at the time of contribution to the plan. B) The actual deferral percentage (ADP) for highly compensated employees in a Section 401(k) plan cannot be more than the ADP of the nonhighly compensated employees multiplied by 1.50. C) These contributions are not subject to federal income tax, FICA, or FUTA. D) These contributions are not subject to payroll taxes but are subject to federal income tax.

A) These contributions are subject to FICA and FUTA but not to federal income tax at the time of contribution to the plan. Employee elective deferrals in Section 401(k) plans are subject to FICA and FUTA but not to federal income tax. These plans are not solely funded by employers. Employee elective deferrals are included.

Tom is a financial planner. Several years ago, as part of his financial planning agreement with Grace, a wealthy client, Tom, agreed to calculate and coordinate the required minimum distributions (RMDs) from Grace's 401(k). Grace continued to work because she owns 3.5% of her employer. Last year, she turned 73 and retired. She deferred her initial RMD payment into this year to avoid adding it to her last year of wages. She took her initial RMD on April 1 of this year. Which of the following statements regarding Grace's RMDs is CORRECT? A) Tom should advise Grace she will need to execute 2 RMDs this year. B) If she rolled the 401(k) into an IRA, Grace could elect NUA tax treatment to lower the overall tax due for each RMD if her account holds any former employer securities. C) By deferring the first RMD to April 1, this year, all subsequent RMDs must also be executed by April 1 each year. D) If Grace does not currently need the RMD income on April 1, this year due to her wealth, she may extend the deferral to December 31.

A) Tom should advise Grace she will need to execute 2 RMDs this year. Because Grace deferred her initial RMD to April 1 of this year, she must execute an additional RMD for the current year prior to December 31. RMDs for subsequent years must be taken by December 31 each year, not April 1. The initial RMD may not be deferred any later than April 1 this year. NUA tax treatment is not available for distributions from an IRA.

All of the following are reasons an employer might favor a nonqualified deferred compensation plan over a qualified retirement plan EXCEPT A) nonqualified plans provide the employer with an immediate income tax deduction. B) nonqualified deferred compensation plans are not subject to all the reporting and disclosure requirements that pertain to qualified retirement plans. C) nonqualified plans do not have to comply with the nondiscrimination rules that apply to qualified plans. D) a nonqualified plan typically has lower administrative costs.

A) nonqualified plans provide the employer with an immediate income tax deduction. Nonqualified deferred compensation plans provide deferred employer deductions and do not allow an income tax deduction until the employee recognizes the income on his tax return.

Lisa is 40 years old and recently applied for a $20,000 life insurance policy. Unknown by her financial planner, she stated that she was 35 years old on her life insurance application. The premium for a 35 year old is $15 per $1,000, which resulted in an annual premium of $300. Had Lisa not misrepresented her age, the premium would have been $25 per $1,000 resulting in an annual premium of $500 for the same policy. Lisa dies unexpectedly 1 year later at age 41. Assuming the insurance company discovers that she misstated her age on the application, what amount will be paid to Lisa's beneficiary/beneficiaries? A) $0 B) $12,000 C) $10,000 D) $19,800

B) $12,000 Because Lisa misrepresented her age, her beneficiary will only receive the amount of death benefit that the annual $300 premium would purchase at her actual age, 40. The amount of death benefit will be adjusted to reflect the correct cost per $1,000 for age 40. Using the correct cost of $25 per $1,000, the face amount will be adjusted to $12,000 ($300 ÷ $25 × $1,000).

MSA stock has a current annual dividend of $0.76 per share, a market price of $23.73 per share, and a beta of 1.47. The current dividend is expected to grow for 3 years at a rate of 3.5% and then 2% thereafter. Assume an investor has a required return of 7%. What is the intrinsic value of MSA stock? A) $29.09 B) $16.17 C) $41.38 D) $26.01

B) $16.17 Based on the multistage growth dividend discount growth model, the intrinsic value of MSA stock is $16.17. Step 1: Compute the value of each future dividend until the growth rate stabilizes (Years 1-3). D1 = $0.7600 × 1.035 = $0.7866 D2 = $0.7866 × 1.035 = $0.8141 D3 = $0.8141 × 1.035 = $0.8426 Step 2: Use the constant growth dividend discount model to calculate the remaining intrinsic value of the stock at the beginning of the year when the dividend growth rate stabilizes (Year 4). D4 = $0.8426 × 1.02 = $0.8595 V = $0.8595 ÷ (0.07 − 0.02) = $17.1896 Step 3: Use the uneven cash flow method to solve for the net present (intrinsi value of the stock. CF0 = 0 CF1 = 0.7866 CF2 = 0.8141 CF3 = 0.8426 + 17.1896 = 18.0322 I/YR = 7% Solve for NPV = 16.1659, or $16.17 (rounded)

In the current year, Garrett invested $100,000 for a 20% partnership interest in an activity in which he is a material participant. The partnership reported a loss of $400,000 in the current year and $200,000 in the next year. Garrett's share of the partnership's loss was $80,000 in the current year and $40,000 in the next year. How much of the loss from the partnership can Garrett deduct? A) $0 in the current year, $0 in the next year. B) $80,000 in the current year, $20,000 in the next year. C) $50,000 in the current year, $50,000 in the next year. D) $80,000 in the current year, $40,000 in the next year.

B) $80,000 in the current year, $20,000 in the next year. Garrett can deduct $80,000 in the current year and $20,000 in the next year. Garrett has $100,000 of capital at risk. Under the at-risk rules he cannot deduct more than $100,000 against his other income. This means he can deduct all of his share of the loss ($80,000) in the current year. In the next year, Garrett can only deduct $20,000 and his nondeductible loss of $20,000 is suspended and can be carried over to be used when the at-risk provisions allow.

During the current year, Mr. Kabel made gifts of the following items to his son: - A bond with an adjusted basis of $12,000 and a fair market value of $40,000. - Stock with an adjusted basis of $22,000 and a fair market value of $33,000. - An auto with an adjusted basis of $12,000 and a fair market value of $14,000. - An interest-free loan of $6,000 for a computer (personal use) on January 1 of the current year; the loan was repaid by the son on December 31 of the current year. Assume the applicable federal rate was 8% per annum. What is the gross amount of gifts includible on Mr. Kabel's gift tax return for the current year? A) $46,000. B) $87,000. C) $52,000. D) $46,480.

B) $87,000. The gross amount of gifts included on Mr. Kabel's gift tax return for the current year is: $40,000 bonds + $33,000 stocks + $14,000 auto = $87,000. Gifts loans of $10,000 or less are not subject to gift tax unless the donee uses the proceeds to purchase income-producing property.

Barbara, a CFP® professional, has been providing comprehensive financial planning services to Lionel for 25 years. During their annual meeting, Lionel discloses to Barbara that he has been sued and may be liable for a substantial judgment that will exceed his insurance coverage. Barbara suggests that Lionel may need to file for Chapter 7 bankruptcy, under which some of his debts will be dischargeable. Which of the following items will be discharged if Lionel files for Chapter 7 bankruptcy? 1. Credit card balances 2. Personal loans 3. Debt due to intentional tort claims 4. Student loans A) 1, 2, and 4 B) 1 and 2 C) 1 only D) 3 and 4

B) 1 and 2 Credit card balances and personal loans can be discharged under Chapter 7 bankruptcy. Debts and obligations that are not dischargeable under Chapter 7 bankruptcy include: intentional tort claims, child support, student loans, government loans, and recent federal income taxes due.

Which of the following statements regarding bonds is(are) CORRECT? 1. Lower coupon bonds are more volatile than higher coupon bonds as interest rates change. 2. Bond prices and changes in interest rates have an inverse relationship. 3. A direct relationship exists between a bond's coupon rate and duration. A) 2 and 3 B) 1 and 2 C) 1, 2, and 3 D) 1 and 3

B) 1 and 2 Lower coupon bonds are more volatile than higher coupon bonds as interest rates change. When interest rates fall, bond prices increase and vice versa. Bond prices and the direction of market interest rates have an inverse relationship. The greater the bond's duration, the greater the price volatility of the bond. The coupon rate and duration of a bond have an inverse relationship.

Which of the following statements regarding private annuities is(are) CORRECT? 1. The transfer of property under a private annuity does not triggers immediate recognition of all capital gain to the seller. 2. If the present value of the annuity payable to the seller is at least equal to the fair value of the property transferred, there is no gift and, as a result, no gift tax due. 3. Each annuity payment will generally consist of a partial return of basis and ordinary income. 4. Private annuities cannot be secured by collateral. A) 1 and 2 B) 1, 2, 3, and 4 C) 2 and 4 D) 1, 2, and 3

B) 1, 2, 3, and 4 All of these statements are correct.

Which of the following statements regarding nonqualified stock options (NQSOs) and incentive stock options (ISOs) is(are) CORRECT? 1. NQSOs will not create an AMT adjustment upon exercise, but ISOs will. 2. Gain may be included in W-2 wages upon exercise for NQSOs but not ISOs. 3. Once vested, both NQSOs and ISOs can be exercised and the stock can be sold immediately. 4. Typically, on the date of the option grant, W-2 compensation income, which is a type of ordinary income, is created for an NQSO but not for an ISO. A) 3 and 4. B) 1, 2, and 3. C) 1 and 2. D) 2, 3, and 4.

B) 1, 2, and 3. Statement 4 is incorrect. The grant date of the option is not typically a taxable event for either an NQSO or an ISO.

Which of the following property would be included in a decedent's probate estate? 1. Solely owned securities held in a brokerage account. 2. An interest in property held as tenants in common with a brother of the decedent. 3. Life insurance policy death proceeds made payable to the decedent's estate. 4. A condo owned jointly (JTWROS) with the decedent's spouse. A) 1 and 2. B) 1, 2, and 3. C) 1 only. D) 2 and 3.

B) 1, 2, and 3. The probate estate includes property that passes by will or by intestate succession, but it does not include property that passes by contract, by law, or under the terms of a trust. In statement 4, the condo was owned with his spouse as JTWROS, which means the condo passes to the surviving spouse by operation of law and is excluded from the probate process.

Which of the following statements regarding universal life (UL) insurance policies is(are) CORRECT? 1. UL policies generally give the policyowner an option to adjust the death benefit. 2. Option A is known as the level death benefit option. 3. UL policies do not allow the policyowner to adjust the premiums. 4. Option B is known as the increasing death benefit option. A) 1, 3, and 4 B) 1, 2, and 4 C) 1 and 2 D) 3 only

B) 1, 2, and 4 Universal life insurance policies offer options to adjust the death benefit and the premiums to help clients meet their financial goals. Option A is known as the level death benefit option because the death benefit only includes the face amount. Option B is known as the increasing death benefit option because the death benefit equals the cash value plus the face amount of the policy.

Which of the following retirement plans must provide for both a qualified preretirement survivor annuity and a qualified joint and survivor annuity for a married plan participant? 1. Traditional defined benefit pension plan. 2. Target benefit pension plan. 3. Profit-sharing plan. 4. Cash balance pension plan. A) 1, 2, 3, and 4. B) 1, 2, and 4. C) 3 and 4. D) 1 and 2.

B) 1, 2, and 4. Pension plans must provide for both a qualified preretirement survivor annuity and a qualified joint and survivor annuity for a married participant. Generally, a profit-sharing plan does not provide a QPSA or QJSA.

Mark, a financial professional, has been providing financial planning services to Peter for 25 years. Peter is widowed and has an adult son who lives in the same city. When Mark began his professional relationship with Peter, Peter was 50 years old and mentally sharp. Recently, however, Mark has noticed that Peter sometimes seems confused during their meetings and is unable to remember details about his financial status. Despite his confusion, Peter has asked Mark to update his estate plan and to make major changes to several key provisions of his will. Which of the following actions is(are) appropriate for Mark in this situation? 1. Advise Peter's son of Peter's requests concerning the estate plan and ask him to accompany Peter on any future meetings. 2. Advise Peter on the importance of appointing someone, such as his son or an attorney-in-fact, to assist him in handling his affairs. 3. Refer Peter to other professionals, such as medical providers or eldercare specialists, who may be able to help Peter address his diminished capacity. A) 2 only B) 2 and 3 C) 1, 2, and 3 D) 3 only

B) 2 and 3 Statements 2 and 3 are correct. By referring Peter to other professionals to address his capacity issues and counseling him on the importance of appointing someone to assist him in handling his affairs, Mark satisfies his fiduciary duty to act in Peter's best interests. Statement 1 is incorrect. Advising Peter's son of Peter's requests concerning his estate plan and asking him to accompany Peter on future meetings violates confidentiality.

Which of the following statements regarding Coverdell Education Savings Accounts (CESAs) are CORRECT? 1. Contributions to a CESA are tax-deductible. 2. Distributions from a CESA are tax-free if used for qualified education expenses. 3. The ability to make CESA contributions is phased out at higher levels of modified adjusted gross income (MAGI). 4. Contributions to a CESA are limited to $2,000 per year per child. A) 1 and 3 B) 2, 3, and 4 C) 1 and 4 D) 1, 2, 3, and 4

B) 2, 3, and 4 Contributions to a CESA are not tax-deductible.

Jeff and Kay, ages 67 and 65, respectively, filed a joint income tax return for the current year. They provided all of the support for their 18-year-old son, who had $2,200 of gross income. Their 23-year-old daughter was a full-time student until her graduation on June 25 of the current year. Before her graduation, she earned $4,450, which was 40% of her total support for the current year. Her parents provided the balance of her support. Jeff and Kay also provided 100% of the support for Kay's father, who is a lifelong resident and citizen of Colombia. How many dependents may Jeff and Kay list on their income tax return for the current year? A) 4. B) 2. C) 3. D) 5.

B) 2. Jeff and Kay can list 2 dependents: 1 for each child. The 23-year old daughter is included because she was a full-time student for almost 6 months. Kay's father is not included because he is not a U.S. citizen (nor a resident of Canada or Mexico) and therefore, not a qualifying person.

Portfolio A has a standard deviation of 12%, and the market has a standard deviation of 16%. The correlation between Portfolio A and the market is 0.5. What percentage of the total risk is unsystematic risk? A) 25% B) 75% C) 50% D) 0%

B) 75% The coefficient of determination (R-squared) of Portfolio A is 25% (0.25). This is calculated by squaring the correlation coefficient (R) of 0.50 (0.50 × 0.50 = 0.25). Therefore, 25% is the percentage of Portfolio A returns that may be explained by the market (or systematic risk). The remainder of the percentage of returns (movement) of Portfolio A is explained by factors independent of the market (or unsystematic risk). To determine this, subtract the systematic risk from 1.0 (1.0 − 0.25 = 0.75).

ABC Inc., a C corporation, has 3 employees: Nick (age 50), Gail (age 25), and Julie (age 25). Nick owns 60% of ABC stock, and Julie and Gail each own 20%. This is a new venture, but cash flows are expected to be relatively stable. The objective of the company is to design a qualified retirement plan that provides a large contribution for Nick with reasonable plan costs and some flexibility in terms of cash commitment. The company expects to contribute approximately $35,000 per year to the retirement plan. Employee Salary Nick $100,000 Julie $30,000 Gail $30,000 Total $160,000 Which of the following plans best meets these objectives? A) Traditional defined benefit pension plan B) Age-based profit-sharing plan C) Cash balance pension plan D) Profit-sharing plan

B) Age-based profit-sharing plan Either profit-sharing plan can provide funding flexibility, but an age-based profit-sharing plan can provide a larger contribution for Nick, as it favors older employees.

Ben has the following securities in his portfolio: - ABC common stock - XYZ common stock - PQR mutual fund (small cap) - DEZ mutual fund (foreign stocks) - 30-year Treasury bond - 5-year Treasury note Ben does not need to be concerned with which of the following risks? A) Systematic risk B) Default risk C) Financial risk D) Reinvestment rate risk

B) Default risk Default risk is the risk that the bond issuer will be unable to service its debt. However, government (Treasury) bonds, unlike corporate and municipal bonds, lack default risk. Investment in common stock is without default risk, because the issuing corporation is not contractually bound to pay dividends. The common stocks and the stock mutual funds in Ben's portfolio fall into this area. All of the other risks apply to the investments in his portfolio.

George, a financial professional, is scheduled to meet with Tim and Jodi this afternoon for a follow-up meeting. In their first meeting, took steps to understand Tim and Jodi's personal and financial circumstances. At this time, George learned that Tim and Jodi have 3 young children and are currently without life insurance coverage. Tim mentioned that life insurance needs have been on his mind lately due to his lack of adequate coverage and his family history of diabetes and heart disease. In addition, George also discovered that the couple owes a substantial amount on their current mortgage. What should George do next? A) George should recommend that they purchase a convertible term life insurance policy for the appropriate face amount of coverage because of Tim's family history. B) George should spend time analyzing Tim and Jodi's qualitative and quantitative information to assess their personal and financial circumstances. C) George should recommend a decreasing term life insurance policy for both Tim and Jodi to make sure the mortgage is taken care of if one of them should die prematurely. D) Because George has already analyzed Tim and Jodi's current course of action and potential alternative course(s) of action, he is ready to communicate his recommendations to the couple.

B) George should spend time analyzing Tim and Jodi's qualitative and quantitative information to assess their personal and financial circumstances. Although George may ultimately recommend that Tim and Jodi purchase a convertible term life insurance policy to help protect their children's financial future or a decreasing term life insurance policy to cover the mortgage, George has not completed the necessary steps to reach the point of recommending financial solutions. George must analyze Tim and Jodi's financial circumstances, current course of action, and potential alternative course(s) of action before he makes recommendations.

Megan has a financial planning practice in Atlanta, Georgia. She is meeting with her clients, Mason and Della Sinclair, to present them with a financial plan she has developed for them. Mason and Della had communicated to Megan that one of their goals is to follow their families' southern tradition and throw their pre-teen daughter, Dixie, a very expensive debutante party in four years. Megan, having grown up in the Midwest, does not understand the purpose of these celebrations and believes that paying for the party is financially irresponsible. What is the best way for Megan to proceed with her clients? A) Because this is not a good use of the Sinclairs' money, Megan should not include it in their plan as a goal. B) Megan should advise the Sinclairs about the impact this goal will have on their overall financial plan. C) Megan should offer Mason and Della her opinion regarding the extravagance of the debutante party. D) Megan should offer alternatives that would be less expensive.

B) Megan should advise the Sinclairs about the impact this goal will have on their overall financial plan. In this case, Megan should understand that debutante parties are part of the culture in which Mason and Della were raised, and following the tradition with Dixie is important to them. Megan should neither voice her opinion on the party, nor should she fail to address this goal in their financial plan. Instead, Megan should let Mason and Della know how such a goal would impact their overall financial plan and the plan's recommendations.

Richard, who is single, has a traditional IRA that contains a portfolio of mutual funds with a current value of $107,000. He is expected to incur approximately $9,600 in medical bills in the next couple of months. Assuming he qualifies to maintain and fund a health savings account (HSA) and he has not made any contributions to the HSA in the current year, could Richard simply transfer the $9,600 from his IRA into an HSA and pay his medical bills with qualified tax-free distributions from his HSA? A) Yes, Richard could make a one-time trustee-to-trustee transfer of $9,600 from his traditional IRA to his HSA. B) No, this type of transfer is not allowed by the Tax Code. C) No. Richard could make a one-time, trustee-to-trustee transfer from his traditional IRA to his HSA, but the estimated medical expenses of $9,600 exceed the maximum allowable contribution for the current year to an HSA for an insured with single coverage; therefore, Richard's transfer is limited to the maximum contribution allowable. D) Yes, Richard could make a one-time trustee-to-trustee transfer of the maximum amount of $15,000 from his traditional IRA to his HSA.

B) No, this type of transfer is not allowed by the Tax Code. An individual can make a one-time, trustee-to-trustee transfer from a traditional IRA to an HSA. This transfer is not subject to income tax but it is limited to the maximum HSA contribution in the year of transfer ($3,650 for an individual in 2022).

Which of the following strategies is consistent with a belief in the efficient market hypothesis? A) Searching for undervalued securities. B) Selecting a random set of stocks for a portfolio. C) Using fundamental analysis to compare the calculated value of a security to the market value of the stock. D) Attempting to predict the overall direction of the securities market.

B) Selecting a random set of stocks for a portfolio. The efficient market hypothesis (EMH) suggests that investors are unable to outperform the market on a consistent basis. The fundamental assumption of this theory is that current stock prices reflect all available information and that stock prices rapidly (or immediately) adjust to reflect any new information. In addition, any new information must be unexpected; therefore, any changes in the stock price resulting from this new information will be random. If prices move in a random fashion, any investment strategies or market techniques designed to take advantage of market inefficiencies are theoretically useless.

Bond A is selling for $1,103.19 and pays a $50 coupon every 6 months. What would happen to the price of this 30-year bond if interest rates rose 2%? A) The price of the bond would change to $774.51. B) The price of the bond would decrease by $190.44. C) The price of the bond would decrease by $328.68. D) The price of the bond would change to $1,374.17.

B) The price of the bond would decrease by $190.44. Market interest rates and bond prices have an inverse relationship. When interest rates go up, bond prices go down. If the current value of the 30-year bond is $1,103.19, then the prevailing market interest rate is 9% (PV = 1,103.19; N = 60; PMT = 50; FV = 1,000; solve for I/YR = 4.5 × 2 = 9.0%). If the interest rate increased 2 percentage points to 11%, the value of the bond would decrease to $912.75 (FV = 1,000; N = 60; I/YR = 5.5(11% ÷ 2); PMT = 50; solve for PV = 912.75, or $912.75). The difference equals $190.44 ($1,103.19 − $912.75).

Stockwell Corporation wants to establish a qualified retirement plan and has identified the following criteria: - Simplicity. - Integration with Social Security. - Flexible funding. - Ability to invest primarily in company stock. - In-service withdrawals. - Distribution of benefits in cash if desired. - Deductible employer contributions. Which of the following types of qualified plans would best meet Stockwell's criteria? A) Target benefit pension plan B) Traditional profit-sharing plan C) Money purchase pension plan D) Employee stock ownership plan (ESOP)

B) Traditional profit-sharing plan Only a traditional profit-sharing plan will meet all of Stockwell's plan objectives. Pension plans limit investment in employer securities and do not offer flexible funding. An ESOP cannot be integrated with Social Security.

Recently, Vance inherited a large amount of one stock from his late Aunt Carol. Vance's basis in the stock is $500,000. He is concerned that the stock may decline in value in the near future. What is the best investment strategy that he could use to protect the stock from substantial downside risk? A) Write a call option B) Zero-cost collar C) Purchase a put option D) Stock index futures

C) Purchase a put option Vance should purchase a put option to protect his position. His portfolio would be protected from downside risk, and his loss is limited to the amount of premium paid for the put option.

In the current year, Donna, who is single, made the following gifts: - $16,000 in medical bills for her sister Robin, with payment made directly to Robin's health care providers. - $12,000 to her mother for rent, utilities, and food. - $10,000 to her cousin, Pat, so he could start a business. - An interest-free demand loan of $30,000 to her cousin, Pat, on January 1 of the current year; the loan was still outstanding at the end of the year. The applicable federal interest rate for that year remained constant at 2%. Pat had no net investment income during the year of the loan. What is the amount of Donna's taxable gifts in the current year? A) $38,000. B) $22,000. C) $0. D) $10,000.

C) $0. The amount of Donna's taxable gifts in the current year is $0. The $16,000 payment of medical bills for her sister is excluded because it was made directly to the health care providers. The $12,000 gift paid to her mother and the $10,000 gift to her cousin are both less than the annual exclusion amount so neither results in a taxable gift. Because the interest-free loan is less than $100,000, and Pat had no net investment income (less than $1,000), the imputed interest is not a gift.

In the current year, Orlando gave $100,000 to his son and $100,000 to his daughter. Orlando's wife also gave $5,000 to his son. No other gifts were made in the current year. Orlando and his wife made the election to split gifts for the year. What is the amount of gifts made by Orlando in the current year? A) $200,000 B) $205,000 C) $102,500 D) $100,000

C) $102,500 Because Orlando and his wife agree to split the gifts, their total gifts are combined and divided by 2 ($205,000 total gifts divided by 2 = $102,500 each).

Kevin wishes to purchase a yacht in 20 years when he retires. If the yacht currently costs $450,000 and inflation is 2% annually, how much should he deposit at the beginning of each year to have enough to purchase the yacht at the end of 20 years? Assume that Kevin will earn an average compounded after-tax annual return of 5% on his investments. A) $20,222 B) $19,620 C) $19,260 D) $21,600

C) $19,260 Future Cost of Yacht PV = 450,000 N = 20 I/YR = 2 PMT = 0 FV = (668,676), or $668,676 Yearly Deposit (BEG Mode) PV = 0 N = 20 I/YR = 5 FV = 668,676 PMTAD = (19,259.52). or $19,260

Robert has a net worth of $200,000 before any of the following transactions: - Paid off credit cards of $10,000 using funds from his savings account. - Transferred $4,000 from his checking account to his Roth IRA. - Purchased $2,000 of furniture on a store credit card. What is Robert's net worth after these transactions? A) $202,000 B) $210,000 C) $200,000 D) $190,000

C) $200,000 None of these transactions will change Robert's net worth. Payment of the credit cards will reduce debt by $10,000. However, using the savings account to pay off the debt will reduce assets by $10,000. The net effect on net worth is zero. The transfer from the checking account reduces assets by $4,000, and the transfer into the Roth IRA increases assets by $4,000. Thus, the net effect on net worth is zero. The addition of the furniture will increase Robert's assets by $2,000. However, the use of credit will increase his debt by $2,000. Therefore, the net effect on net worth is zero.

A calendar-year taxpayer made the following charitable contributions in the current year: Contribution Basis Fair Market Value Cash to church $5,000 $5,000 Unimproved land to the city of Kenner, Louisiana $40,000 $70,000 The land had been held as an investment and was acquired 5 years ago. Shortly after receipt, the city of Kenner sold the land for $90,000. If the taxpayer's AGI is $120,000, the maximum allowable charitable contribution deduction in the current year is: A) $25,000 if the basis deduction election is not made. B) $36,000 if the basis deduction election is not made. C) $45,000 if the basis deduction election is made. D) $75,000 if the basis deduction election is made.

C) $45,000 if the basis deduction election is made. The maximum allowable charitable deduction for the current year is $45,000 if the basis deduction election is made. The taxpayer has 2 options with respect to the contribution deduction. First, she can deduct the FMV of the land limited to 30% of her AGI. In this case, the total deduction would be $41,000 [$5,000 cash + $36,000 ($70,000 FMV limited to 30% of $120,000)]. The carryover for the next 5 years is $34,000 ($70,000 FMV − $36,000 current year deduction). Alternatively, she can deduct the adjusted basis, limited to 50% of AGI. The total deduction would be $40,000 for the land and $5,000 for the cash (cash contributions to 50% organizations are limited to 60% of AGI, well over the $5,000), for a total of $45,000. There would be no carryover with this option but the deduction for the current year would me maximized.

Sharon, single and age 58, retired two years ago and is receiving a $600 monthly pension from her previous employer's qualified pension plan. She recently accepted a position at a small CPA firm that has no pension plan. She will receive $5,000 in annual compensation from the CPA firm and will continue to receive $7,200 in annual pension benefits. What is Sharon's maximum deductible contribution to a traditional IRA for 2022? A) $6,000 B) $0 C) $5,000 D) $7,000

C) $5,000 Sharon is over age 50 and does not participate in an employer-sponsored retirement plan. Therefore, she can contribute the lesser of her earned income ($5,000) or $7,000 to a traditional IRA in 2022.

Randy, age 40 and not a key employee, has employer-provided group term life insurance equal to twice his salary of $75,000. He makes a monthly contribution to pay for the insurance of $5. Randy's wife Pamela is the sole beneficiary. The employer's actual cost for Randy's life insurance protection is $0.25 per month per $1,000 of death benefit, and the uniform premium for group term under the Internal Revenue Code is $0.10 per month per $1,000 of death benefit. What annual amount must Randy report for federal income tax purposes as a result of his group term insurance benefit? A) $120 B) $0 C) $60 D) $204

C) $60 Group term life insurance premiums up to the first $50,000 of face value paid by the employer are tax exempt for the employee. For any amount of group term life insurance coverage greater than $50,000, the scheduled premium per $1,000 per the Uniform Premium Table is included in the employee's W-2 income. Excess coverage (in thousands) 100 IRC cost per thousand × 0.10 Monthly cost of coverage $10 Multiply monthly cost by 12 for annual cost. Annual cost of coverage (12 × $10) = $120 Less: Randy's contribution (12 × $5) = $60 Taxable amount = $60

Lance, age 60, has been investing $350 at the end of each month in a tax-deferred variable annuity for the last 10 years. The annuity has been earning a compound rate of return of 6%. If Lance withdraws all of the money from his annuity today, will he have enough money after paying taxes at 12% to purchase a motor home for $56,000? A) Yes, because he has $59,200 to purchase the motor home. B) Yes, because he has $57,358 to purchase the motor home. C) No, because he has only $55,515 to purchase the motor home. D) No, because he has only $48,754 to purchase the motor home.

C) No, because he has only $55,515 to purchase the motor home. Step 1: Calculate value today PV = 0 N = 120 (10 × 12) I/YR = 0.5 (6 ÷ 12) PMT = (350) FV = 57,357.77, or $57,357.77 Step 2: Determine Sam's basis Amount of payments $350 Number of payments × 120 Basis $42,000 Step 3: Calculate taxable amount Proceeds $57,357.77 Less: basis (42,000.00) Gain $15,357.77 Step 4: Cash available Proceeds $57,357.77 Less: tax (1,842.93) or ($15,357.77 × 0.12) After-tax cash $55,514.84,rounded to $55,515

Edwin, age 83, is a wealthy investor with a net worth of $12.5 million. He wants to begin gifting his assets to friends and family members, as long as he can do so without incurring any generation-skipping transfer tax (GSTT) or using any of his lifetime GSTT exemption. The people to whom he would like to make gifts include the following: - His ex-wife, Lisa, age 40 - His former daughter-in-law, Sheila, age 35, who was married to his son, Doug, until two years ago - His granddaughter, Claire, age 22 All of Edwin's children are still living. Which of the following recommendations will achieve Edwin's objectives of avoiding any GSTT and not using any of his lifetime GSTT exemption? 1. Make a cash gift of $100,000 to Lisa 2. Make a cash gift of $100,000 to Sheila 3. Make a cash gift of $100,000 to Claire A) 1 and 3 B) 2 only C) 1 and 2 D) None of these recommendations will achieve Edwin's objectives

C) 1 and 2 Statement 1 is correct; a transferor's spouse or former spouse is not a skip person, regardless of age, so a gift to Lisa has no GSTT consequences. Statement 2 is correct; a spouse or former spouse of the transferor's lineal descendant is assigned to the same generation as the lineal descendant, so Sheila is the same generation as Edwin's son and is not a skip person. Statement 3 is incorrect. Because all of Edwin's children are still living, Claire is a skip person (i.e., the predeceased parent rule does not apply) and a gift to her is subject to the GSTT. After applying the $16,000 annual exclusion (for 2022), a gift of $100,000 to Claire will result in a taxable GSTT transfer of $84,000.

Generally speaking, which of the following property is subject to probate? 1. Property owned outright in one's own name at the time of death (fee simple). 2. An interest in property held as tenants in common. 3. Life insurance and other death proceeds payable to the decedent's estate. 4. The decedent's half of any community property. A) 2 and 4 B) 3 and 4 C) 1, 2, 3, and 4 D) 1 and 2

C) 1, 2, 3, and 4 The probate estate includes all property that passes by testate or intestate succession. This includes property in the decedent's name alone, the decedent's interest in property held as tenants in common, and insurance proceeds payable to the estate. The probate estate also includes the decedent's half of any community property.

Based on Markowitz's theory, which of the following portfolios do NOT belong on the efficient frontier? Portfolio ExpectedReturn StandardDeviation 1 10% 12% 2 11% 13% 3 14% 12% 4 17% 17% 5 19% 17%

C) 1, 2, and 4 The efficient frontier consists of portfolios with the highest expected return for a given level of risk. Portfolio 3 has a higher expected return and a standard deviation less than or equal to portfolios 1 and 2. Therefore, portfolios 1 and 2 are not on the efficient frontier. Portfolio 5 has a higher expected return and a standard deviation equal to portfolio 4. Therefore, portfolio 4 is not on the efficient frontier.

Duane, a financial planner, is meeting with Lisa, who wants information regarding how several investment sales she has completed this year will impact her income tax return. Lisa has sold the following properties: Section 1202 stock, which Lisa purchased on December 12, 2019, for $50,000 and sold on December 15, 2022, for its FMV of $65,000. A vacation home Lisa inherited from her uncle, who died in 2020 and who had a basis of $95,000 in the home. The home had a FMV of $135,000 in his gross estate. Lisa sold it for $160,000 on July 1, 2022. Lisa has used the vacation home only four weeks since she inherited it; otherwise it was vacant. Stock Lisa inherited from another uncle, who also died in 2020. His basis in the stock was $20,000 and the FMV in his gross estate was $15,000. Lisa sold it on November 1, 2022, for $17,000. What should Duane tell Lisa? 1. 100% of the gain on the sale of the Section 1202 stock is excluded from Lisa's income for both regular income tax and AMT purposes. 2. Lisa must recognize a $2,000 gain on the sale of the stock she inherited from her uncle. 3. Lisa's gain on the vacation home is all capital gain. 4. Lisa must recognize a total gain of $27,000 on the sale of her investments. A) 2, 3, and 4 B) 1 only C) 2 and 3 D) 1, 2, and 4

C) 2 and 3 Statement 2 is correct. Lisa's basis in the stock is the FMV of the stock in her uncle's gross estate, $15,000. Statement 3 is correct. All of the gain on the sale of the vacation home is capital gain. Statement 1 is incorrect. Section 1202 stock must be held for five years in order for Lisa to exclude the gain from her taxable income. Statement 4 is incorrect. The total gain Lisa must recognize is $42,000 ($15,000 on the Section 1202 stock + $25,000 on the vacation home sale + $2,000 from the stock sale). Note that the vacation home does NOT qualify for the Section 121 gain exclusion, which applies to the sale of a personal residence only.

James, 32, and Coralie, 30, have been married for 7 years and have no children. Recently, both James and Coralie were promoted to management positions at a downtown marketing firm and now have a significant amount of disposable income. The couple lives in the suburbs and they are planning to purchase a loft style apartment downtown within the next 6 months. Ideally, James and Coralie would like to invest their money in a safe place for the down payment during the 6 months they intend to spend searching for the perfect location and amenities. Which of the following investments would you recommend to James and Coralie to accomplish this goal? A) Stock Index Fund B) Investment-Grade Bond Fund C) Money Market Fund D) U.S. Government Income Fund

C) Money Market Fund James and Coralie are preparing to make a significant purchase within a relatively short period of time. They will require a highly liquid investment to keep their money safe. Money market funds provide a modest rate of return, diversification, and liquidity. Based on these characteristics, the money market fund best matches the couple's investment objective.

Beauty Co. employs 200 nonexcludable employees, 20 of whom are highly compensated. Sixteen of the 20 highly compensated and 125 of the 180 non-highly compensated employees benefit from the Beauty Co. qualified pension plan. The average benefits accrued for the highly compensated is 8%. For the non-highly compensated, the average accrued benefit is 3%. Which of the following statements best describes the plan's coverage? A) The plan does not meet the ratio percentage test or the average benefits test. B) The plan does not meet the ratio percentage test. C) The plan meets the ratio percentage test. D) Although the plan does not meet the ratio percentage test, it meets the average benefits test.

C) The plan meets the ratio percentage test. The plan meets the ratio percentage test. The percentage of highly-compensated employees covered by the plan is 80% (16 ÷ 20). The percentage of non-highly compensated employees covered by the plan is 69% (125 ÷ 180). The plan exceeds the 70% required by the ratio percentage test (69% ÷ 80% = 86%). The plan fails the average benefits test (3% ÷ 8% = 37.5%).

An 80-year-old widower explains to you that he is risk averse and wishes to find an investment that will provide him with preservation of capital. Which of the following should you recommend? A) preferred stock B) long-term U.S. government bonds C) bank-insured CDs D) S&P 500 index fund

C) bank-insured CDs The best answer is the bank-insured CDs. Certificates of deposit (CDs) are deposits made with a bank or savings and loan for a specified period, commonly one month to five years. CDs have traditionally been used to provide an income stream to retirees. CDs are FDIC insured, which is often a reason for investors' interest in purchasing CDs.

Bill and Curt have entered into an exchange of real property. Bill gives Curt unimproved land that has an FMV of $15,000. In exchange, Curt forgives a $20,000 debt that Bill owes him. Bill acquired the land in an estate auction for a bargain price of $6,000. What is Bill's realized gain in this transaction? A) $9,000 B) $15,000 C) $6,000 D) $14,000

D) $14,000 The gain realized is the difference between the amount realized and the adjusted basis. Bill had a total cost basis in the land of $6,000. The FMV of the land when he gave it to Curt was $15,000. If Bill had instead sold the land at that time, he would have had a net gain of $9,000. In addition to this gain, he had a $20,000 debt forgiven for the price of the $15,000 unimproved land. In addition to the net gain of $9,000, he also received an additional $5,000 in non-cash consideration (boot) because of the debt forgiveness that was in excess of the FMV of the land, for a total gain of $14,000.

Mr. Ortego died on December 29 of last year. The assets in his estate were valued on his date of death and alternate valuation date as follows: Asset Date of Death Valuation Alternate Valuation Residence $750,000 $700,000 Common stock $16,400,000 $16,450,000 Municipal bonds $180,000 $150,000 Patent $80,000 $65,000 The patent had 5 years of life remaining at the time of Mr. Ortego's death. The executor sold the residence on March 1 of this year for $725,000. If Mr. Ortego's executor properly elects to use the alternate valuation date, what is the value of Mr. Ortego's gross estate? A) $17,365,000 B) $17,380,000 C) $17,390,000 D) $17,405,000.

D) $17,405,000. The alternate valuation date is 6 months from the date of death. When using the alternate valuation date, the election applies to all assets, with 2 exceptions. One exception is for wasting assets, such as patents, annuities, and installment notes, which must be valued at the date of death. The second exception is for assets disposed of after the date of death, but before the alternate valuation date. These assets are valued as of the date of disposal. Therefore, the calculation would be: residence $725,000 + stock $16,450,000 + bonds $150,000 + patent $80,000 = $17,405,000.

Until yesterday, Fred was employed by Avias Company. The company has a group health insurance plan that covers 24 employees. Fred's health coverage included his spouse and 2 dependent children, ages 5 and 7. Which of the following events would qualify Fred, his spouse, or his dependents for COBRA coverage and for how long? 1. Fred's death (36 months) 2. Termination of employment as a result of dismissal due to downsizing (36 months) 3. Divorce (29 months) 4. Legal separation (36 months) A) 1, 2, and 3 B) 1, 2, and 4 C) 3 and 4 D) 1 and 4

D) 1 and 4 Qualifying events include death of the covered employee, termination of employment, including retiring, voluntary resignation, being laid off, and being fired for anything except gross misconduct, a change in status (e.g., full-time to part-time), divorce or legal separation causing the spouse and/or dependent children to lose coverage, child reaching an age where the child is no longer eligible to be covered, employee reaching Medicare age, and spouse and/or dependent child losing coverage as a result. Voluntary or involuntary termination qualifies the individual for 18 months of COBRA coverage. Divorce, legal separation, or termination of employment due to death qualifies the individual for 36 months of COBRA coverage.

Which of the following statements describe basic provisions of a money purchase pension plan? 1. As a defined contribution plan, a money purchase plan is not subject to the minimum funding standard. 2. The employer may deduct for a plan contribution up to a maximum of 25% of covered payroll. 3. The employer contribution generally is allocated based on relative compensation. 4. Forfeitures from nonvested participants' accounts must be applied to reduce the employer contribution. A) 1 and 2 B) 3 and 4 C) 1, 2, and 4 D) 2 and 3

D) 2 and 3 The employer deduction limit for a money purchase plan is 25% of covered payroll, and employer contributions generally are allocated based on relative compensation. A money purchase plan is a "pension" plan and thus is subject to the minimum funding requirements. As in any defined contribution plan, the plan may provide for forfeitures either to be reallocated to remaining participants' accounts or applied to reduce the employer contribution.

Nick, age 38, earns $250,000 per year from a personally owned regular C corporation. He wants to establish a defined contribution plan. His 3 employees, each of whom earn $20,000, are between ages 26 and 30, and have been employed with the company for an average of 4 years. Which of the following vesting schedules would be the most appropriate for Nick's qualified retirement plan from the employer's perspective? A) 3-year cliff. B) 3-7 year graduated vesting. C) 5-year cliff. D) 2-6 year graduated vesting.

D) 2-6 year graduated vesting. Defined contribution plans must vest at least as rapidly as 3-year cliff vesting or 2-6 year graded vesting. However, 3-year cliff vesting must provide 100% vesting after 3 years. Two-six year graduated vesting provides for 20% vesting at 2 years and increases 20% per year to 100% by year 6. Given the average tenure of Nick's employees is 4 years, from an employer's perspective the most advantageous vested schedule is 2-6 year graded.

Louie, a representative for a large pharmaceutical corporation, would like to purchase a home. Up to this point, he has been renting a condo in fear that he may have to relocate. A recent promotion has Louie working out of corporate headquarters and he has received reassurance from senior management that he will not be relocated for at least three to five years. Based on this information, he would like to own a place of his own. If Louie finds the right home, which mortgage should he choose? A) 30-year fixed FHA B) None of these choices are appropriate C) 15-year fixed conventional D) 5-year adjustable rate mortgage

D) 5-year adjustable rate mortgage If Louie expects to be in the home for a short time, he should consider an adjustable rate mortgage (ARM). Generally, an ARM has a lower interest rate than 30-year conventional mortgages. The downside risk of an ARM is that increases in interest rates may cause the mortgage payment to increase.

Assume that an investor has earned the following series of returns on an investment: Year Return 1 18.3% 2 0.7% 3 −7.6% 4 11.9% 5 2.5% Which row correctly identifies the investor's arithmetic and geometric mean? Arithmetic Mean Geometric Mean A) 8.20% 8.01% B) 8.01% 5.16% C) 4.77% 8.20% D) 5.16% 4.77%

D) 5.16% 4.77% Arithmetic mean (18.3% + 0.7% − 7.6% + 11.9% + 2.5%) ÷ 5 = 5.16% Geometric mean (Assume an investment of $1) PV = −1 FV = (1 + 0.183)(1 + 0.007)(1 − 0.076)(1 + 0.119)(1 + 0.025) = 1.2625 N = 5 Solve for I/YR = 4.7727, or 4.77%

BCD Corporation is in the process of promoting a new bond issue to a wealthy potential private investor. The bonds will feature a 5.5% coupon, a par value of $1,000, a 30-year maturity, and an A rating. The issue is callable in 10 years for 103. Assuming the bond will be priced at par, which of the following statements is(are) CORRECT? I. The bond has a yield to call of 5.73%. II. The bond's call price is $1,030. A) I only B) II only C) Neither I nor II D) Both I and II

D) Both I and II Both statements I and II are correct. The call price is 103% of $1,000 or $1,030. Yield to call: PV = −1,000 FV = 1,030 PMT = 27.50 ($55.00 ÷ 2) N = 20 Solve for I/YR = 2.8632 × 2 = 5.7264% (rounded to 5.73%)

Janice, age 50, wants an investment that will offer her the opportunity for long-term growth with moderate risk. She wants a customized portfolio based on an asset-based fee structure. Which of the following would be the best choice for Janice? A) Balanced mutual fund. B) Growth and income mutual fund. C) S&P 500 Index exchange-traded fund. D) Large-cap growth separately managed account.

D) Large-cap growth separately managed account. The best choice for Janice is the large-cap growth separately managed account. This type of investment account offers a customized portfolio approach and an asset-based fee structure for portfolio management services.

Dana, CEO of One Last Time Fitness, has a salary of $100,000 and was awarded the following stock options from her company: Stock Option Grant Date Type Exercise Price # Shares A 6/1/18 ISO $7 200 B 1/1/19 ISO $17 100 C 4/1/21 ISO $27 300 D 5/1/18 ISO $37 200 E 1/1/19 ISO $47 200 Consider the following transactions regarding the above stock options: Stock Option Event Date Action # Shares Market Price on Action Date A 1/1/20 Exercised 200 $17 B 4/2/21 Exercised 100 $27 A 9/15/22 Sold 200 $57 D 7/11/22 Exercised 200 $77 B 12/8/22 Sold 100 $37 C 12/15/22 Exercised 300 $47 C 12/15/22 Sold 300 $67 What are the ordinary income, capital gain, and AMT tax consequences resulting from Dana liquidating position B on 12/8/22? A) Short-term capital gain of $2,000, positive AMT adjustment of $1,000 B) Ordinary income of $2,000, negative AMT adjustment of $1,000 C) Long-term capital gain of $1,000, negative AMT adjustment of $1,000 D) Long-term capital gain of $2,000, negative AMT adjustment of $1,000

D) Long-term capital gain of $2,000, negative AMT adjustment of $1,000 Dana satisfied the ISO requirements for receiving favorable tax treatment on the sale of position B on 12/8/2022. If the stock acquired by exercise of the ISO is not sold until after one year from the date of the option's exercise and two years from the date of its grant, any gain in the value of the stock is treated as a long-term capital gain to the employee. She will also have a negative AMT adjustment of $1,000. Whenever a position created from an ISO is sold, it will be subject to a negative AMT adjustment. Because the bargain element was $1,000 when B was exercised [100 shares × ($27 − $17) = $1,000], the adjustment will be negative $1,000 when the position is sold. Dana will also have a $2,000 long term capital gain [100 shares × ($37 − $17) = $2,000] as a result of liquidating position B.

Joan gives her son property with an adjusted tax basis of $35,000 and a fair market value of $30,000. No gift tax is paid. Joan's son subsequently sells the property for $33,000. What is his recognized gain (or loss)? A) $2,000 loss. B) $3,000 gain. C) $33,000 gain. D) No gain or loss.

D) No gain or loss. Because the fair market value (FMV) on the date of the gift was less than the donor's (Joan's) adjusted tax basis, the double basis rule applies. Under the double basis rule, no gain or loss is recognized if the donee sells the property at a price that is between the donor's adjusted basis and the FMV on the date of the gift.

Following a seminar, a young, single client approaches a financial planner with $12,000 and states that she would like to develop a financial plan and invest in the stock market. This is her first experience investing and she would like help choosing an appropriate account. What is the financial planner's most appropriate course of action? A) Determine whether the client has any consumer debt B) Recommend suitable investments C) Open and fund a Roth IRA for the current year D) Open a brokerage account

D) Open a brokerage account Of the answer options provided, reviewing debt is the best course of action. The financial planner needs additional information from the client before taking an action involving increasing client risk, such as opening a brokerage account. Additional information is also needed before making investment recommendations. The financial planner does not have enough information to determine if a Roth IRA is appropriate.

Jack, age 36, is a self-made millionaire and enjoys taking financial risks. He would like to retire by age 60. He has amassed a sizable investment portfolio, but he is concerned the asset allocation is not maximizing growth. He is willing to take risks to achieve higher long-term returns. Which of the following portfolios would be most suitable for Jack to meet his retirement goal? A) Portfolio 1 - 20% S&P 500 Index Fund, 50% Small-Cap Growth Fund, 25% International Stock Fund. B) Portfolio 3 - 40% Small-Cap Growth Fund, 20% Corporate Bond Fund, 15% Utilities Stock Fund, 15% Foreign Stock Fund, 10% Municipal Bond Fund. C) Portfolio 2 - 50% Russell 2000 Index Fund, 20% Corporate Bond Fund, 20% Biotechnology Fund, 10% Energy Sector Fund. D) Portfolio 4 - 60% S&P 500 Index Fund, 10% Small-Cap Growth Fund, 10% Foreign Stock Fund, 10% Pacific Rim Growth Fund, 10% Corporate Bond Fund.

D) Portfolio 4 - 60% S&P 500 Index Fund, 10% Small-Cap Growth Fund, 10% Foreign Stock Fund, 10% Pacific Rim Growth Fund, 10% Corporate Bond Fund. Portfolio 4 is the best choice for Jack based on his age and risk tolerance. A mix of 90% stocks and 10% bonds is appropriate for an investor who is willing to take risks to achieve higher long-term returns.

Mary Anne is considering the purchase of Davidson stock. The stock has a market price of $35 and is currently paying an annual dividend of $1.75 per share, which grows by a constant 4% each year. On the basis of the constant growth dividend discount model, is this a good purchase for her portfolio if Mary Anne's required return is 9%? A) Yes. The stock is overvalued by $1.40 per share. B) No. The stock is overvalued by $1.40 and, therefore, should not be purchased. C) This is an adequate purchase because the calculated value equals the current market price. D) Yes. The stock is undervalued by $1.40 per share.

D) Yes. The stock is undervalued by $1.40 per share. According to the constant growth dividend discount model, the stock is undervalued by $1.40 ($36.40 − $35). V=D1/r−g V=$1.75(1+0.04)/0.09−0.04=$1.82/0.05=$36.40

Which of the following does the Federal Reserve use to control the money supply? 1. Adjusting the discount rate 2. Open market operations 3. Fiscal policy A) 2 only B) 2 and 3 C) 1 only D) 1 and 2

D) 1 and 2 The Federal Reserve can control the money supply by adjusting the discount rate. For example, a higher discount rate discourages banks from lending money and reduces the amount of money in circulation. The Federal Reserve also uses open market operations to control the money supply. By buying government securities in the open market, for example, the Fed can increase the amount of money in circulation. Fiscal policy is conducted by Congress.


Related study sets

Concepts in Federal Taxation - Chapter 1

View Set

Mucoskeletal system (60, 61, 62, 63)

View Set

Finance chapter 6 conceptual questions

View Set